Switching order of integration on unbounded domain












2












$begingroup$


Let's say ${fleft( {x,y} right)}$ is a continuous function and assume that:



$intlimits_{ - infty }^infty {left| {fleft( {x,y} right)} right|dx} $ converges for all $y $



2) $intlimits_{ - infty }^infty {left| {fleft( {x,y} right)} right|dy} $ converges for all $x $



Is the following statement correct?
$$intlimits_{ - infty }^infty {intlimits_{ - infty }^infty {fleft( {x,y} right)dxdy} } = intlimits_{ - infty }^infty {intlimits_{ - infty }^infty {fleft( {x,y} right)dydx} } $$



This is supposed to be some version of Fubini's or Tonelli's theorem but I wasn't able to find the exact version for this case.










share|cite|improve this question











$endgroup$












  • $begingroup$
    It's not clear what follows "assume that:".
    $endgroup$
    – Rigel
    Dec 23 '18 at 19:08










  • $begingroup$
    I'm sorry, I meant to say they converge. I'll edit it
    $endgroup$
    – zokomoko
    Dec 23 '18 at 19:11










  • $begingroup$
    This can be treated as a corollary to normal integral switching in double integrals. By following that, you should see the limits don't change in this case (or do they? try and see). But in the case of only one unbounded, you could feel like you're in trouble. I suggest you consider a finite 'infinity' as they do I optical theory and follow this: youtube.com/watch?v=j6A44yQrGfU related: math.stackexchange.com/questions/467910/…
    $endgroup$
    – Nick
    Dec 23 '18 at 19:22
















2












$begingroup$


Let's say ${fleft( {x,y} right)}$ is a continuous function and assume that:



$intlimits_{ - infty }^infty {left| {fleft( {x,y} right)} right|dx} $ converges for all $y $



2) $intlimits_{ - infty }^infty {left| {fleft( {x,y} right)} right|dy} $ converges for all $x $



Is the following statement correct?
$$intlimits_{ - infty }^infty {intlimits_{ - infty }^infty {fleft( {x,y} right)dxdy} } = intlimits_{ - infty }^infty {intlimits_{ - infty }^infty {fleft( {x,y} right)dydx} } $$



This is supposed to be some version of Fubini's or Tonelli's theorem but I wasn't able to find the exact version for this case.










share|cite|improve this question











$endgroup$












  • $begingroup$
    It's not clear what follows "assume that:".
    $endgroup$
    – Rigel
    Dec 23 '18 at 19:08










  • $begingroup$
    I'm sorry, I meant to say they converge. I'll edit it
    $endgroup$
    – zokomoko
    Dec 23 '18 at 19:11










  • $begingroup$
    This can be treated as a corollary to normal integral switching in double integrals. By following that, you should see the limits don't change in this case (or do they? try and see). But in the case of only one unbounded, you could feel like you're in trouble. I suggest you consider a finite 'infinity' as they do I optical theory and follow this: youtube.com/watch?v=j6A44yQrGfU related: math.stackexchange.com/questions/467910/…
    $endgroup$
    – Nick
    Dec 23 '18 at 19:22














2












2








2


1



$begingroup$


Let's say ${fleft( {x,y} right)}$ is a continuous function and assume that:



$intlimits_{ - infty }^infty {left| {fleft( {x,y} right)} right|dx} $ converges for all $y $



2) $intlimits_{ - infty }^infty {left| {fleft( {x,y} right)} right|dy} $ converges for all $x $



Is the following statement correct?
$$intlimits_{ - infty }^infty {intlimits_{ - infty }^infty {fleft( {x,y} right)dxdy} } = intlimits_{ - infty }^infty {intlimits_{ - infty }^infty {fleft( {x,y} right)dydx} } $$



This is supposed to be some version of Fubini's or Tonelli's theorem but I wasn't able to find the exact version for this case.










share|cite|improve this question











$endgroup$




Let's say ${fleft( {x,y} right)}$ is a continuous function and assume that:



$intlimits_{ - infty }^infty {left| {fleft( {x,y} right)} right|dx} $ converges for all $y $



2) $intlimits_{ - infty }^infty {left| {fleft( {x,y} right)} right|dy} $ converges for all $x $



Is the following statement correct?
$$intlimits_{ - infty }^infty {intlimits_{ - infty }^infty {fleft( {x,y} right)dxdy} } = intlimits_{ - infty }^infty {intlimits_{ - infty }^infty {fleft( {x,y} right)dydx} } $$



This is supposed to be some version of Fubini's or Tonelli's theorem but I wasn't able to find the exact version for this case.







multivariable-calculus






share|cite|improve this question















share|cite|improve this question













share|cite|improve this question




share|cite|improve this question








edited Dec 23 '18 at 19:57







zokomoko

















asked Dec 23 '18 at 19:06









zokomokozokomoko

169214




169214












  • $begingroup$
    It's not clear what follows "assume that:".
    $endgroup$
    – Rigel
    Dec 23 '18 at 19:08










  • $begingroup$
    I'm sorry, I meant to say they converge. I'll edit it
    $endgroup$
    – zokomoko
    Dec 23 '18 at 19:11










  • $begingroup$
    This can be treated as a corollary to normal integral switching in double integrals. By following that, you should see the limits don't change in this case (or do they? try and see). But in the case of only one unbounded, you could feel like you're in trouble. I suggest you consider a finite 'infinity' as they do I optical theory and follow this: youtube.com/watch?v=j6A44yQrGfU related: math.stackexchange.com/questions/467910/…
    $endgroup$
    – Nick
    Dec 23 '18 at 19:22


















  • $begingroup$
    It's not clear what follows "assume that:".
    $endgroup$
    – Rigel
    Dec 23 '18 at 19:08










  • $begingroup$
    I'm sorry, I meant to say they converge. I'll edit it
    $endgroup$
    – zokomoko
    Dec 23 '18 at 19:11










  • $begingroup$
    This can be treated as a corollary to normal integral switching in double integrals. By following that, you should see the limits don't change in this case (or do they? try and see). But in the case of only one unbounded, you could feel like you're in trouble. I suggest you consider a finite 'infinity' as they do I optical theory and follow this: youtube.com/watch?v=j6A44yQrGfU related: math.stackexchange.com/questions/467910/…
    $endgroup$
    – Nick
    Dec 23 '18 at 19:22
















$begingroup$
It's not clear what follows "assume that:".
$endgroup$
– Rigel
Dec 23 '18 at 19:08




$begingroup$
It's not clear what follows "assume that:".
$endgroup$
– Rigel
Dec 23 '18 at 19:08












$begingroup$
I'm sorry, I meant to say they converge. I'll edit it
$endgroup$
– zokomoko
Dec 23 '18 at 19:11




$begingroup$
I'm sorry, I meant to say they converge. I'll edit it
$endgroup$
– zokomoko
Dec 23 '18 at 19:11












$begingroup$
This can be treated as a corollary to normal integral switching in double integrals. By following that, you should see the limits don't change in this case (or do they? try and see). But in the case of only one unbounded, you could feel like you're in trouble. I suggest you consider a finite 'infinity' as they do I optical theory and follow this: youtube.com/watch?v=j6A44yQrGfU related: math.stackexchange.com/questions/467910/…
$endgroup$
– Nick
Dec 23 '18 at 19:22




$begingroup$
This can be treated as a corollary to normal integral switching in double integrals. By following that, you should see the limits don't change in this case (or do they? try and see). But in the case of only one unbounded, you could feel like you're in trouble. I suggest you consider a finite 'infinity' as they do I optical theory and follow this: youtube.com/watch?v=j6A44yQrGfU related: math.stackexchange.com/questions/467910/…
$endgroup$
– Nick
Dec 23 '18 at 19:22










2 Answers
2






active

oldest

votes


















1












$begingroup$

I think the conclusion is false.



Let $gcolon mathbb{R}tomathbb{R}$ be a continuous non-negative function, such that
$g(x) = 0$ if $xnotin [0,1]$, and $int_0^1 g = 1$.
Let us consider the function
$$
f(x,y) = sum_{k=0}^infty [g(x-k) - g(x-k-1)] g(y-k),
qquad (x,y)inmathbb{R}^2.
$$

Observe that, for every $(x,y)inmathbb{R}^2$, at most one term of the series is different from $0$, and that assumptions 1) and 2) are satisfied.



Since, if $xin [n, n+1]$, $ninmathbb{N}$, one has
$$
f(x,y) =
begin{cases}
g(x)g(y), & text{if} n = 0,
\
g(x-n)[g(y-n) - g(y-n-1)],
& text{if} ngeq 1,
end{cases}
$$

it is easy to see that
$$
int_{-infty}^{+infty} dx int_{-infty}^{+infty} f(x,y), dy = 1.
$$

On the other hand, if $yin[n,n+1]$,
$$
f(x,y) = [g(x-n) - g(x-n-1)] g(y-n),
$$

hence
$int_{-infty}^{+infty} f(x,y) dx = 0$ for every $yinmathbb{R}$, and
$int_{-infty}^{+infty} dy int_{-infty}^{+infty} f(x,y) dx = 0$ as well.






share|cite|improve this answer









$endgroup$





















    0












    $begingroup$

    There's something of a standard library of analysis counterexamples. Looking here... iterated integrals don't match, but the inner integrals are unconditional... OK, I've got it. Well, the version I have in memory is for sums: $begin{bmatrix}1&0&0&0&cdots\-1&1&0&0&cdots\0&-1&1&0&cdots\0&0&-1&1&cdots\ vdots&vdots&vdots&vdots&ddotsend{bmatrix}$



    Sum along the rows first (inner) and we get $1$. Sum along the columns first and we get zero.



    This can of course be converted into a version for the integral. Let
    $$f(x,y)=begin{cases}1& 0<x-y<1\ -1&-1<x-y<0\0&text{otherwise}end{cases}$$
    for $x,yge 0$. Then
    begin{align*}int_0^{infty}f(x,y),dx &= begin{cases}1-y& 0le yle 1\ 0&y>1end{cases}\
    int_0^{infty}f(x,y),dy &= begin{cases}x-1& 0le xle 1\ 0&x>1end{cases}end{align*}

    so $int_0^{infty}int_0^{infty}f(x,y),dx,dy = frac12$ and $int_0^{infty}int_0^{infty}f(x,y),dy,dx = -frac12$. The order of the integral cannot be interchanged in general.



    Now, quibbles. That's not continuous! No problem; just smooth the jumps out when defining $f$ - say, one period of a sawtooth wave $f(x,y)=g(x-y)$ where $g(t)=begin{cases}t&-frac12le tle frac12\1-t&frac12le tle 1\-1-t&-1le tle -frac12\ 0&text{otherwise}end{cases}$. Calculating the integral is a little messier, but we'll still get something positive for $int_0^{infty} f(x,y),dx$ when $yin (0,1)$ and something negative for $int_0^{infty} f(x,y),dy$ when $xin (0,1)$.

    That's an integral on $[0,infty)^2$, not $mathbb{R}^2$! Padding the function with zeros when either variable is negative would do the trick - except that it disrupts continuity again. So then, we need to tweak it so that it's zero on the axes. OK - multiply $f$ by $min{x,y,1}$. Working this out for my sawtooth example, it's a lot of calculation for not much benefit, so I'll just give the answer:
    $$int_0^{infty} f(x,y),dx=begin{cases}0& yge 2\ frac{(2-y)^3}{6}& frac32le yle 2\ frac{y(frac32-y)^2}{6}+frac{5-3y}{24}& 1le yle frac32\ frac y4 - frac{3y-1}{24} - frac{(2-y)(y-frac12)^2}{6} & frac12 le yle 1\ frac y4-frac{y^3}{6}&0le yle frac12\ 0&text{otherwise}end{cases}$$
    All but one of those are clearly nonnegative; we simplify that third case to clear the last bit up:
    begin{align*}frac y4 - frac{3y-1}{24} - frac{(2-y)(y-frac12)^2}{6} &= frac{4y-3y+1-4y^3+12y^2-9y+2}{24}\
    &= frac{-4y^3 + 12y^2 - 8y + 1}{24}\
    &= frac1{24}-frac{y(1-y)(2-y)}{6}\
    frac y4 - frac{3y-1}{24} - frac{(2-y)(y-frac12)^2}{6} &ge frac1{24}-frac{(1-y)(2-y)}{6}ge 0end{align*}

    (The inequalities, of course, use that $frac12 le yle 1$)
    So then, $int_0^{infty} f(x,y),dx$ is always nonnegative. By the symmetry of $f$'s definition, $int_0^{infty} f(x,y),dy$ is always nonpositive, and the two iterated integrals are unequal.



    This example runs on essentially the same principles as Rigel's already posted answer, but I find it a little clearer to work with the explicit piecewise definition than the fake infinite sum.






    share|cite|improve this answer









    $endgroup$














      Your Answer








      StackExchange.ready(function() {
      var channelOptions = {
      tags: "".split(" "),
      id: "69"
      };
      initTagRenderer("".split(" "), "".split(" "), channelOptions);

      StackExchange.using("externalEditor", function() {
      // Have to fire editor after snippets, if snippets enabled
      if (StackExchange.settings.snippets.snippetsEnabled) {
      StackExchange.using("snippets", function() {
      createEditor();
      });
      }
      else {
      createEditor();
      }
      });

      function createEditor() {
      StackExchange.prepareEditor({
      heartbeatType: 'answer',
      autoActivateHeartbeat: false,
      convertImagesToLinks: true,
      noModals: true,
      showLowRepImageUploadWarning: true,
      reputationToPostImages: 10,
      bindNavPrevention: true,
      postfix: "",
      imageUploader: {
      brandingHtml: "Powered by u003ca class="icon-imgur-white" href="https://imgur.com/"u003eu003c/au003e",
      contentPolicyHtml: "User contributions licensed under u003ca href="https://creativecommons.org/licenses/by-sa/3.0/"u003ecc by-sa 3.0 with attribution requiredu003c/au003e u003ca href="https://stackoverflow.com/legal/content-policy"u003e(content policy)u003c/au003e",
      allowUrls: true
      },
      noCode: true, onDemand: true,
      discardSelector: ".discard-answer"
      ,immediatelyShowMarkdownHelp:true
      });


      }
      });














      draft saved

      draft discarded


















      StackExchange.ready(
      function () {
      StackExchange.openid.initPostLogin('.new-post-login', 'https%3a%2f%2fmath.stackexchange.com%2fquestions%2f3050619%2fswitching-order-of-integration-on-unbounded-domain%23new-answer', 'question_page');
      }
      );

      Post as a guest















      Required, but never shown

























      2 Answers
      2






      active

      oldest

      votes








      2 Answers
      2






      active

      oldest

      votes









      active

      oldest

      votes






      active

      oldest

      votes









      1












      $begingroup$

      I think the conclusion is false.



      Let $gcolon mathbb{R}tomathbb{R}$ be a continuous non-negative function, such that
      $g(x) = 0$ if $xnotin [0,1]$, and $int_0^1 g = 1$.
      Let us consider the function
      $$
      f(x,y) = sum_{k=0}^infty [g(x-k) - g(x-k-1)] g(y-k),
      qquad (x,y)inmathbb{R}^2.
      $$

      Observe that, for every $(x,y)inmathbb{R}^2$, at most one term of the series is different from $0$, and that assumptions 1) and 2) are satisfied.



      Since, if $xin [n, n+1]$, $ninmathbb{N}$, one has
      $$
      f(x,y) =
      begin{cases}
      g(x)g(y), & text{if} n = 0,
      \
      g(x-n)[g(y-n) - g(y-n-1)],
      & text{if} ngeq 1,
      end{cases}
      $$

      it is easy to see that
      $$
      int_{-infty}^{+infty} dx int_{-infty}^{+infty} f(x,y), dy = 1.
      $$

      On the other hand, if $yin[n,n+1]$,
      $$
      f(x,y) = [g(x-n) - g(x-n-1)] g(y-n),
      $$

      hence
      $int_{-infty}^{+infty} f(x,y) dx = 0$ for every $yinmathbb{R}$, and
      $int_{-infty}^{+infty} dy int_{-infty}^{+infty} f(x,y) dx = 0$ as well.






      share|cite|improve this answer









      $endgroup$


















        1












        $begingroup$

        I think the conclusion is false.



        Let $gcolon mathbb{R}tomathbb{R}$ be a continuous non-negative function, such that
        $g(x) = 0$ if $xnotin [0,1]$, and $int_0^1 g = 1$.
        Let us consider the function
        $$
        f(x,y) = sum_{k=0}^infty [g(x-k) - g(x-k-1)] g(y-k),
        qquad (x,y)inmathbb{R}^2.
        $$

        Observe that, for every $(x,y)inmathbb{R}^2$, at most one term of the series is different from $0$, and that assumptions 1) and 2) are satisfied.



        Since, if $xin [n, n+1]$, $ninmathbb{N}$, one has
        $$
        f(x,y) =
        begin{cases}
        g(x)g(y), & text{if} n = 0,
        \
        g(x-n)[g(y-n) - g(y-n-1)],
        & text{if} ngeq 1,
        end{cases}
        $$

        it is easy to see that
        $$
        int_{-infty}^{+infty} dx int_{-infty}^{+infty} f(x,y), dy = 1.
        $$

        On the other hand, if $yin[n,n+1]$,
        $$
        f(x,y) = [g(x-n) - g(x-n-1)] g(y-n),
        $$

        hence
        $int_{-infty}^{+infty} f(x,y) dx = 0$ for every $yinmathbb{R}$, and
        $int_{-infty}^{+infty} dy int_{-infty}^{+infty} f(x,y) dx = 0$ as well.






        share|cite|improve this answer









        $endgroup$
















          1












          1








          1





          $begingroup$

          I think the conclusion is false.



          Let $gcolon mathbb{R}tomathbb{R}$ be a continuous non-negative function, such that
          $g(x) = 0$ if $xnotin [0,1]$, and $int_0^1 g = 1$.
          Let us consider the function
          $$
          f(x,y) = sum_{k=0}^infty [g(x-k) - g(x-k-1)] g(y-k),
          qquad (x,y)inmathbb{R}^2.
          $$

          Observe that, for every $(x,y)inmathbb{R}^2$, at most one term of the series is different from $0$, and that assumptions 1) and 2) are satisfied.



          Since, if $xin [n, n+1]$, $ninmathbb{N}$, one has
          $$
          f(x,y) =
          begin{cases}
          g(x)g(y), & text{if} n = 0,
          \
          g(x-n)[g(y-n) - g(y-n-1)],
          & text{if} ngeq 1,
          end{cases}
          $$

          it is easy to see that
          $$
          int_{-infty}^{+infty} dx int_{-infty}^{+infty} f(x,y), dy = 1.
          $$

          On the other hand, if $yin[n,n+1]$,
          $$
          f(x,y) = [g(x-n) - g(x-n-1)] g(y-n),
          $$

          hence
          $int_{-infty}^{+infty} f(x,y) dx = 0$ for every $yinmathbb{R}$, and
          $int_{-infty}^{+infty} dy int_{-infty}^{+infty} f(x,y) dx = 0$ as well.






          share|cite|improve this answer









          $endgroup$



          I think the conclusion is false.



          Let $gcolon mathbb{R}tomathbb{R}$ be a continuous non-negative function, such that
          $g(x) = 0$ if $xnotin [0,1]$, and $int_0^1 g = 1$.
          Let us consider the function
          $$
          f(x,y) = sum_{k=0}^infty [g(x-k) - g(x-k-1)] g(y-k),
          qquad (x,y)inmathbb{R}^2.
          $$

          Observe that, for every $(x,y)inmathbb{R}^2$, at most one term of the series is different from $0$, and that assumptions 1) and 2) are satisfied.



          Since, if $xin [n, n+1]$, $ninmathbb{N}$, one has
          $$
          f(x,y) =
          begin{cases}
          g(x)g(y), & text{if} n = 0,
          \
          g(x-n)[g(y-n) - g(y-n-1)],
          & text{if} ngeq 1,
          end{cases}
          $$

          it is easy to see that
          $$
          int_{-infty}^{+infty} dx int_{-infty}^{+infty} f(x,y), dy = 1.
          $$

          On the other hand, if $yin[n,n+1]$,
          $$
          f(x,y) = [g(x-n) - g(x-n-1)] g(y-n),
          $$

          hence
          $int_{-infty}^{+infty} f(x,y) dx = 0$ for every $yinmathbb{R}$, and
          $int_{-infty}^{+infty} dy int_{-infty}^{+infty} f(x,y) dx = 0$ as well.







          share|cite|improve this answer












          share|cite|improve this answer



          share|cite|improve this answer










          answered Dec 23 '18 at 19:54









          RigelRigel

          11.4k11320




          11.4k11320























              0












              $begingroup$

              There's something of a standard library of analysis counterexamples. Looking here... iterated integrals don't match, but the inner integrals are unconditional... OK, I've got it. Well, the version I have in memory is for sums: $begin{bmatrix}1&0&0&0&cdots\-1&1&0&0&cdots\0&-1&1&0&cdots\0&0&-1&1&cdots\ vdots&vdots&vdots&vdots&ddotsend{bmatrix}$



              Sum along the rows first (inner) and we get $1$. Sum along the columns first and we get zero.



              This can of course be converted into a version for the integral. Let
              $$f(x,y)=begin{cases}1& 0<x-y<1\ -1&-1<x-y<0\0&text{otherwise}end{cases}$$
              for $x,yge 0$. Then
              begin{align*}int_0^{infty}f(x,y),dx &= begin{cases}1-y& 0le yle 1\ 0&y>1end{cases}\
              int_0^{infty}f(x,y),dy &= begin{cases}x-1& 0le xle 1\ 0&x>1end{cases}end{align*}

              so $int_0^{infty}int_0^{infty}f(x,y),dx,dy = frac12$ and $int_0^{infty}int_0^{infty}f(x,y),dy,dx = -frac12$. The order of the integral cannot be interchanged in general.



              Now, quibbles. That's not continuous! No problem; just smooth the jumps out when defining $f$ - say, one period of a sawtooth wave $f(x,y)=g(x-y)$ where $g(t)=begin{cases}t&-frac12le tle frac12\1-t&frac12le tle 1\-1-t&-1le tle -frac12\ 0&text{otherwise}end{cases}$. Calculating the integral is a little messier, but we'll still get something positive for $int_0^{infty} f(x,y),dx$ when $yin (0,1)$ and something negative for $int_0^{infty} f(x,y),dy$ when $xin (0,1)$.

              That's an integral on $[0,infty)^2$, not $mathbb{R}^2$! Padding the function with zeros when either variable is negative would do the trick - except that it disrupts continuity again. So then, we need to tweak it so that it's zero on the axes. OK - multiply $f$ by $min{x,y,1}$. Working this out for my sawtooth example, it's a lot of calculation for not much benefit, so I'll just give the answer:
              $$int_0^{infty} f(x,y),dx=begin{cases}0& yge 2\ frac{(2-y)^3}{6}& frac32le yle 2\ frac{y(frac32-y)^2}{6}+frac{5-3y}{24}& 1le yle frac32\ frac y4 - frac{3y-1}{24} - frac{(2-y)(y-frac12)^2}{6} & frac12 le yle 1\ frac y4-frac{y^3}{6}&0le yle frac12\ 0&text{otherwise}end{cases}$$
              All but one of those are clearly nonnegative; we simplify that third case to clear the last bit up:
              begin{align*}frac y4 - frac{3y-1}{24} - frac{(2-y)(y-frac12)^2}{6} &= frac{4y-3y+1-4y^3+12y^2-9y+2}{24}\
              &= frac{-4y^3 + 12y^2 - 8y + 1}{24}\
              &= frac1{24}-frac{y(1-y)(2-y)}{6}\
              frac y4 - frac{3y-1}{24} - frac{(2-y)(y-frac12)^2}{6} &ge frac1{24}-frac{(1-y)(2-y)}{6}ge 0end{align*}

              (The inequalities, of course, use that $frac12 le yle 1$)
              So then, $int_0^{infty} f(x,y),dx$ is always nonnegative. By the symmetry of $f$'s definition, $int_0^{infty} f(x,y),dy$ is always nonpositive, and the two iterated integrals are unequal.



              This example runs on essentially the same principles as Rigel's already posted answer, but I find it a little clearer to work with the explicit piecewise definition than the fake infinite sum.






              share|cite|improve this answer









              $endgroup$


















                0












                $begingroup$

                There's something of a standard library of analysis counterexamples. Looking here... iterated integrals don't match, but the inner integrals are unconditional... OK, I've got it. Well, the version I have in memory is for sums: $begin{bmatrix}1&0&0&0&cdots\-1&1&0&0&cdots\0&-1&1&0&cdots\0&0&-1&1&cdots\ vdots&vdots&vdots&vdots&ddotsend{bmatrix}$



                Sum along the rows first (inner) and we get $1$. Sum along the columns first and we get zero.



                This can of course be converted into a version for the integral. Let
                $$f(x,y)=begin{cases}1& 0<x-y<1\ -1&-1<x-y<0\0&text{otherwise}end{cases}$$
                for $x,yge 0$. Then
                begin{align*}int_0^{infty}f(x,y),dx &= begin{cases}1-y& 0le yle 1\ 0&y>1end{cases}\
                int_0^{infty}f(x,y),dy &= begin{cases}x-1& 0le xle 1\ 0&x>1end{cases}end{align*}

                so $int_0^{infty}int_0^{infty}f(x,y),dx,dy = frac12$ and $int_0^{infty}int_0^{infty}f(x,y),dy,dx = -frac12$. The order of the integral cannot be interchanged in general.



                Now, quibbles. That's not continuous! No problem; just smooth the jumps out when defining $f$ - say, one period of a sawtooth wave $f(x,y)=g(x-y)$ where $g(t)=begin{cases}t&-frac12le tle frac12\1-t&frac12le tle 1\-1-t&-1le tle -frac12\ 0&text{otherwise}end{cases}$. Calculating the integral is a little messier, but we'll still get something positive for $int_0^{infty} f(x,y),dx$ when $yin (0,1)$ and something negative for $int_0^{infty} f(x,y),dy$ when $xin (0,1)$.

                That's an integral on $[0,infty)^2$, not $mathbb{R}^2$! Padding the function with zeros when either variable is negative would do the trick - except that it disrupts continuity again. So then, we need to tweak it so that it's zero on the axes. OK - multiply $f$ by $min{x,y,1}$. Working this out for my sawtooth example, it's a lot of calculation for not much benefit, so I'll just give the answer:
                $$int_0^{infty} f(x,y),dx=begin{cases}0& yge 2\ frac{(2-y)^3}{6}& frac32le yle 2\ frac{y(frac32-y)^2}{6}+frac{5-3y}{24}& 1le yle frac32\ frac y4 - frac{3y-1}{24} - frac{(2-y)(y-frac12)^2}{6} & frac12 le yle 1\ frac y4-frac{y^3}{6}&0le yle frac12\ 0&text{otherwise}end{cases}$$
                All but one of those are clearly nonnegative; we simplify that third case to clear the last bit up:
                begin{align*}frac y4 - frac{3y-1}{24} - frac{(2-y)(y-frac12)^2}{6} &= frac{4y-3y+1-4y^3+12y^2-9y+2}{24}\
                &= frac{-4y^3 + 12y^2 - 8y + 1}{24}\
                &= frac1{24}-frac{y(1-y)(2-y)}{6}\
                frac y4 - frac{3y-1}{24} - frac{(2-y)(y-frac12)^2}{6} &ge frac1{24}-frac{(1-y)(2-y)}{6}ge 0end{align*}

                (The inequalities, of course, use that $frac12 le yle 1$)
                So then, $int_0^{infty} f(x,y),dx$ is always nonnegative. By the symmetry of $f$'s definition, $int_0^{infty} f(x,y),dy$ is always nonpositive, and the two iterated integrals are unequal.



                This example runs on essentially the same principles as Rigel's already posted answer, but I find it a little clearer to work with the explicit piecewise definition than the fake infinite sum.






                share|cite|improve this answer









                $endgroup$
















                  0












                  0








                  0





                  $begingroup$

                  There's something of a standard library of analysis counterexamples. Looking here... iterated integrals don't match, but the inner integrals are unconditional... OK, I've got it. Well, the version I have in memory is for sums: $begin{bmatrix}1&0&0&0&cdots\-1&1&0&0&cdots\0&-1&1&0&cdots\0&0&-1&1&cdots\ vdots&vdots&vdots&vdots&ddotsend{bmatrix}$



                  Sum along the rows first (inner) and we get $1$. Sum along the columns first and we get zero.



                  This can of course be converted into a version for the integral. Let
                  $$f(x,y)=begin{cases}1& 0<x-y<1\ -1&-1<x-y<0\0&text{otherwise}end{cases}$$
                  for $x,yge 0$. Then
                  begin{align*}int_0^{infty}f(x,y),dx &= begin{cases}1-y& 0le yle 1\ 0&y>1end{cases}\
                  int_0^{infty}f(x,y),dy &= begin{cases}x-1& 0le xle 1\ 0&x>1end{cases}end{align*}

                  so $int_0^{infty}int_0^{infty}f(x,y),dx,dy = frac12$ and $int_0^{infty}int_0^{infty}f(x,y),dy,dx = -frac12$. The order of the integral cannot be interchanged in general.



                  Now, quibbles. That's not continuous! No problem; just smooth the jumps out when defining $f$ - say, one period of a sawtooth wave $f(x,y)=g(x-y)$ where $g(t)=begin{cases}t&-frac12le tle frac12\1-t&frac12le tle 1\-1-t&-1le tle -frac12\ 0&text{otherwise}end{cases}$. Calculating the integral is a little messier, but we'll still get something positive for $int_0^{infty} f(x,y),dx$ when $yin (0,1)$ and something negative for $int_0^{infty} f(x,y),dy$ when $xin (0,1)$.

                  That's an integral on $[0,infty)^2$, not $mathbb{R}^2$! Padding the function with zeros when either variable is negative would do the trick - except that it disrupts continuity again. So then, we need to tweak it so that it's zero on the axes. OK - multiply $f$ by $min{x,y,1}$. Working this out for my sawtooth example, it's a lot of calculation for not much benefit, so I'll just give the answer:
                  $$int_0^{infty} f(x,y),dx=begin{cases}0& yge 2\ frac{(2-y)^3}{6}& frac32le yle 2\ frac{y(frac32-y)^2}{6}+frac{5-3y}{24}& 1le yle frac32\ frac y4 - frac{3y-1}{24} - frac{(2-y)(y-frac12)^2}{6} & frac12 le yle 1\ frac y4-frac{y^3}{6}&0le yle frac12\ 0&text{otherwise}end{cases}$$
                  All but one of those are clearly nonnegative; we simplify that third case to clear the last bit up:
                  begin{align*}frac y4 - frac{3y-1}{24} - frac{(2-y)(y-frac12)^2}{6} &= frac{4y-3y+1-4y^3+12y^2-9y+2}{24}\
                  &= frac{-4y^3 + 12y^2 - 8y + 1}{24}\
                  &= frac1{24}-frac{y(1-y)(2-y)}{6}\
                  frac y4 - frac{3y-1}{24} - frac{(2-y)(y-frac12)^2}{6} &ge frac1{24}-frac{(1-y)(2-y)}{6}ge 0end{align*}

                  (The inequalities, of course, use that $frac12 le yle 1$)
                  So then, $int_0^{infty} f(x,y),dx$ is always nonnegative. By the symmetry of $f$'s definition, $int_0^{infty} f(x,y),dy$ is always nonpositive, and the two iterated integrals are unequal.



                  This example runs on essentially the same principles as Rigel's already posted answer, but I find it a little clearer to work with the explicit piecewise definition than the fake infinite sum.






                  share|cite|improve this answer









                  $endgroup$



                  There's something of a standard library of analysis counterexamples. Looking here... iterated integrals don't match, but the inner integrals are unconditional... OK, I've got it. Well, the version I have in memory is for sums: $begin{bmatrix}1&0&0&0&cdots\-1&1&0&0&cdots\0&-1&1&0&cdots\0&0&-1&1&cdots\ vdots&vdots&vdots&vdots&ddotsend{bmatrix}$



                  Sum along the rows first (inner) and we get $1$. Sum along the columns first and we get zero.



                  This can of course be converted into a version for the integral. Let
                  $$f(x,y)=begin{cases}1& 0<x-y<1\ -1&-1<x-y<0\0&text{otherwise}end{cases}$$
                  for $x,yge 0$. Then
                  begin{align*}int_0^{infty}f(x,y),dx &= begin{cases}1-y& 0le yle 1\ 0&y>1end{cases}\
                  int_0^{infty}f(x,y),dy &= begin{cases}x-1& 0le xle 1\ 0&x>1end{cases}end{align*}

                  so $int_0^{infty}int_0^{infty}f(x,y),dx,dy = frac12$ and $int_0^{infty}int_0^{infty}f(x,y),dy,dx = -frac12$. The order of the integral cannot be interchanged in general.



                  Now, quibbles. That's not continuous! No problem; just smooth the jumps out when defining $f$ - say, one period of a sawtooth wave $f(x,y)=g(x-y)$ where $g(t)=begin{cases}t&-frac12le tle frac12\1-t&frac12le tle 1\-1-t&-1le tle -frac12\ 0&text{otherwise}end{cases}$. Calculating the integral is a little messier, but we'll still get something positive for $int_0^{infty} f(x,y),dx$ when $yin (0,1)$ and something negative for $int_0^{infty} f(x,y),dy$ when $xin (0,1)$.

                  That's an integral on $[0,infty)^2$, not $mathbb{R}^2$! Padding the function with zeros when either variable is negative would do the trick - except that it disrupts continuity again. So then, we need to tweak it so that it's zero on the axes. OK - multiply $f$ by $min{x,y,1}$. Working this out for my sawtooth example, it's a lot of calculation for not much benefit, so I'll just give the answer:
                  $$int_0^{infty} f(x,y),dx=begin{cases}0& yge 2\ frac{(2-y)^3}{6}& frac32le yle 2\ frac{y(frac32-y)^2}{6}+frac{5-3y}{24}& 1le yle frac32\ frac y4 - frac{3y-1}{24} - frac{(2-y)(y-frac12)^2}{6} & frac12 le yle 1\ frac y4-frac{y^3}{6}&0le yle frac12\ 0&text{otherwise}end{cases}$$
                  All but one of those are clearly nonnegative; we simplify that third case to clear the last bit up:
                  begin{align*}frac y4 - frac{3y-1}{24} - frac{(2-y)(y-frac12)^2}{6} &= frac{4y-3y+1-4y^3+12y^2-9y+2}{24}\
                  &= frac{-4y^3 + 12y^2 - 8y + 1}{24}\
                  &= frac1{24}-frac{y(1-y)(2-y)}{6}\
                  frac y4 - frac{3y-1}{24} - frac{(2-y)(y-frac12)^2}{6} &ge frac1{24}-frac{(1-y)(2-y)}{6}ge 0end{align*}

                  (The inequalities, of course, use that $frac12 le yle 1$)
                  So then, $int_0^{infty} f(x,y),dx$ is always nonnegative. By the symmetry of $f$'s definition, $int_0^{infty} f(x,y),dy$ is always nonpositive, and the two iterated integrals are unequal.



                  This example runs on essentially the same principles as Rigel's already posted answer, but I find it a little clearer to work with the explicit piecewise definition than the fake infinite sum.







                  share|cite|improve this answer












                  share|cite|improve this answer



                  share|cite|improve this answer










                  answered Dec 23 '18 at 22:59









                  jmerryjmerry

                  17.1k11633




                  17.1k11633






























                      draft saved

                      draft discarded




















































                      Thanks for contributing an answer to Mathematics Stack Exchange!


                      • Please be sure to answer the question. Provide details and share your research!

                      But avoid



                      • Asking for help, clarification, or responding to other answers.

                      • Making statements based on opinion; back them up with references or personal experience.


                      Use MathJax to format equations. MathJax reference.


                      To learn more, see our tips on writing great answers.




                      draft saved


                      draft discarded














                      StackExchange.ready(
                      function () {
                      StackExchange.openid.initPostLogin('.new-post-login', 'https%3a%2f%2fmath.stackexchange.com%2fquestions%2f3050619%2fswitching-order-of-integration-on-unbounded-domain%23new-answer', 'question_page');
                      }
                      );

                      Post as a guest















                      Required, but never shown





















































                      Required, but never shown














                      Required, but never shown












                      Required, but never shown







                      Required, but never shown

































                      Required, but never shown














                      Required, but never shown












                      Required, but never shown







                      Required, but never shown







                      Popular posts from this blog

                      Plaza Victoria

                      In PowerPoint, is there a keyboard shortcut for bulleted / numbered list?

                      How to put 3 figures in Latex with 2 figures side by side and 1 below these side by side images but in...